Đến nội dung

Sin99 nội dung

Có 237 mục bởi Sin99 (Tìm giới hạn từ 30-04-2020)



Sắp theo                Sắp xếp  

#724118 Đề thi môn Toán chuyên tuyển sinh lớp 10 chuyên Trần Hưng Đạo Bình Thuận năm...

Đã gửi bởi Sin99 on 23-07-2019 - 13:37 trong Tài liệu - Đề thi

 Bài 1. Cho một tập A={a1,a2,a3,a4}A={a1,a2,a3,a4} gồm 4 số nguyên dương phân biệt. Ký hiệu tổng a1+a2+a3+a4a1+a2+a3+a4 bởi sAsA. Đặt nAnA là số các cặp (i;j)(i;j) với 1≤i<j≤41≤i<j≤4 và ai+ajai+aj là ước số của sAsA. Tìm tất cả các tập AA sao cho nAnA đạt giá trị lớn nhất có thể.
C1Bài 1. Ngon.
$s_A=a_1+a_2+a_3+a_4 $Có tất cả 6 cặp $a_i+a_j=a_1+a_2,a_2+a_3,a_3+a_4,a_1+a_4,a_2+a_3,a_ 2+a_4 $
Do 4 số nguyên dương phân biệt nên mình sắp lại thứ tự có $a_1<a_2<a_3<a_4 $.Khi đó $a_3+a_4|s_A $ khi $a_3+a_4|a_1+a_2 $ vô lý,cái $a_2+a_4 $ cũng thế.
Do vậy $n_A\leq 4 $
Có hệ $\begin{cases}
& a_1+a_2|s_A=a_1+a_2+a_3+a_4 \\ 
& a_1+a_3|s_A=a_1+a_2+a_3+a_4\\ 
& a_1+a_4|s_A=a_1+a_2+a_3+a_4\\ 
& a_2+a_3|s_A=a_1+a_2+a_3+a_4
\end{cases}
\rightarrow
\begin{cases}
& a_1+a_2|a_3+a_4 \\ 
& a_1+a_3|a_2+a_4\\ 
& a_1+a_4|a_2+a_3\\ 
& a_2+a_3|a_1+a_4
\end{cases}
\rightarrow
\begin{cases}
& a_1+a_2|a_3+a_4 \\ 
& a_1+a_3|a_2+a_4\\ 
& a_1+a_4=a_2+a_3\\ 
& a_1<a_2<a_3<a_4\\ 
\end{cases}
$
Cái này dễ giải ra tính được 4 số theo 1 số còn lại.Mathlinks ra đáp số ($x,5x,7x,11x $) và $(x,11x,19x,29x) $  
C2Đóng góp bài 1:
Giải : Ta gọi Hai cặp trong là bù nhau nếu tổng bằng $s_a $. 
Thấy rằng Hai cặp bù nhau, hoặc bằng nhau, hoặc chỉ có một trong chúng thuộc $n_a $.
Gs $a_1<a_2<a_3<a_4 $. Thì hoặc $a_1+a_2 $ hoặc $a_3+a_4 $ thuộc $n_a $.
$a_2+a_4 $ và $a_1+a_3 $ cũng vậy.
Do đó $n_a $ có tối đa 4 phần tử.
Điều này xảy ra khi : $a_1+a_4=a_2+a_3 $.
Đặt $a_1=x, a_2= x+d, a_3=y, a_4=y+d $.
Ta có : $a_1+a_2= 2x+d $ chia hết $2x+2y+2d $ (1)
$a_1+a_3= x+y $ chia hết $2d $ nhưng $y> x+d > d $. Nên $x+y =2d $.
(1) suy ra $2x+d $ chia hết $6d $. Vậy $2x+d=3d $ do $x<d $. Vậy $x= d:2 $.
Đáp số là $x, 3x, 5x, 7x $.  

Bài 2. Một tập hữu hạn SS gồm ít nhất 2 điểm trên mặt phẳng. Giả sử không có 3 điểm nào của SS thẳng hàng. Một cối xay gió là một quá trình bắt đầu với một đường thẳng ℓℓ đi qua một điểm duy nhất PSPS. Đường thẳng quay theo chiều kim đồng hồ quanh PP cho đến khi gặp một điểm khác cũng thuộc SS. Điểm mới này, QQ, là trục quay mới, và đường thẳng ℓℓ tiếp tục quay theo chiều kim đồng hồ đến khi gặp một điểm khác của SS. Quá trình này lặp lại vô hạn lần.
Chứng minh rằng ta có thể chọn một điểm PSPS và đường thẳng ℓℓ đi qua PP sao cho mỗi điểm của SS được sử dụng làm trục quay vô hạn lần.
Bài 2:
Ta gọi một đường thằng $d $ đi qua điểm $A $ và một điểm $B $ là tốt với $A $ nếu quay $d $ quanh $A $ một góc $\alpha $ nhỏ (chiều quay kim đồng hồ) thì hai nửa mặt phẳng chứa số điểm chênh lệch nhau không quá $1 $.

Nhận xét 1: Mỗi điểm $A $ thì có ít nhất $1 $ đường thẳng tốt đi qua. 

Bây giờ ta sẽ chứng minh là với cách chọn điểm $A $ bất kì và đường thẳng $ l $ là đường thẳng tốt qua $A $ 

Nhận xét 2: Giả sử tại bước thứ $k $, ta quay tại điểm $A $ và đường thẳng $l $ là đường thẳng tốt thì ta sẽ đến tới điểm $B $, đường thẳng $l $ sẽ trở thành đường thẳng $BA $ và $BA $ là tốt
Chứng minh:

Thật vậy ta giả sử $l $ chứa $2 $ điểm $A $ và $X $( song song với đường ngang). Số điểm nằm ở mặt phẳng phía trên là $a $, phía dưới là $b $. Ta có do $l $ tốt nên $|a+1-b|\le 1 $. Nếu $B $ ở mặt phẳng phía trên thì khi quay đường $BA $ một góc $\alpha $ nhỏ chia mặt phẳng thành hai phần có số điểm chênh nhau là $|a+1-b|\le 1 $. Nếu $B $ ở phía dưới thì cũng dễ thấy khi quay $BA $ quanh $B $ một góc $\alpha $ nhỏ đi thì số điểm chênh lệch ở hai nửa mặt phẳng là $|a+1-b|\le 1 $. Do đó $BA $ là đường thẳng tốt. Nhận xét được chứng minh.

Nhận xét 3: Mỗi điểm $X $ thuộc $S $ sẽ được ghé đến ít nhất một lần không phụ thuộc vào điểm đầu ( và do đó được ghé đến vô hạn lần).

Ta biết rằng quá trình quay đường thẳng $l $ theo chiều kim đồng hồ là một quá trình vô hạn lần và sẽ có lúc đường thẳng $l $ song song với đường thẳng tốt $d $ đi qua $X $ và điểm $M $. Nếu $l $ không trùng $d $, xét điểm cuối cùng mà $l $ đi qua trước khi song song với $X $ là $N,Y $. Dễ thấy là $M,X $ khác với $N,Y $Tuy nhiên dẽ thấy là sẽ có ít nhất một trong hai đường thẳng $NY $ và $MX $ không tốt. ( chỉ cần so sánh số điểm chênh lệch ở các nửa mặt phẳng là ok)

Vậy bài toán được chứng minh  

 

Bài 3. Cho f:R→Rf:R→R thỏa mãn
 

f(x+y)≤yf(x)+f(f(x))f(x+y)≤yf(x)+f(f(x))

với mọi số thực x,yx,y. Chứng minh rằng f(x)=0x≤0f(x)=0x≤0.
C1Bài 3:
[M]f(x+y) \le yf(x)+f(f(x))[/M] (1)
Thay [M]y=0[/M] ta có : [M]f(x) \le f(f(x))[/M]
Kí hiệu [M]D_f[/M] là tập giá trị của [M]f [/M]. Với [M]x \in D_f[/M], thay [M]y= f(x)-x[/M] không âm ta có [M]f(f(x)) \le yf(x)+f(f(x))[/M] nên [M]f(x)[/M] không âm với [M]x \in D_f[/M] nói cách khác [M]f(f(x))[/M] không âm với mọi [M]x [/M].
Bây giờ, giả sử tồn tại [M]x[/M] mà [M]f(x)>0 [/M], cho y tiến tới âm vô cùng thì sẽ tồn tại [M]e[/M] mà [M]f(x)<0[/M] với mọi [M]x \le e[/M]
tương tự nếu có [M]f(x)<0[/M] thì [M]f(x)<0[/M] với mọi [M]x[/M] đủ lớn.
Chú ý rằng nếu [M]f(x)<0[/M] thì [M]x[/M] không thuộc tập giá trị của [M]f[/M]
TH1: [M]f[/M] nhận cả giá trị âm dương. Thì [M]D_f[/M] là tập bị chặn . Mâu thuẫn.
TH2: [M]f[/M] chỉ nhận giá trị dương cũng mâu thuẫn 
Vậy [M]f[/M] chỉ nhận giá trị không dương .
Như thế [M]f(f(x))=0[/M] với mọi [M]x[/M]
suy ra [M]f(0)=f(f(f(x)))=0[/M]
Ta có [M]f(x+y) \le yf(x)[/M]
Cho [M]y=-x[/M] suy ra [M]-xf(x) \ge 0[/M] nên [M]f(x)[/M] không âm với [M]x[/M] không âm 
Suy ra [M]f(x)=0[/M] với [M]x[/M] không âm.

C2 Bài 3: $f(x+y) \leq yf(x)+f(f(x)). $ (*)
+ Cho $y=0 $, có $f(x)\leq f(f(x)) \; $ (1).
+ Cho $y=f(x)-x $, có $f(f(x)) \leq f(x)[f(x)-x]+f(f(x)) $, hay là $0 \leq f(x)[f(x)-x] \; $ (2).
a) Ta sẽ chứng minh $f(x)\leq 0 $ với mọi $x $. Thật vậy, nếu có $x $ sao cho $f(x)>0 $ thì
$f(f(y))=f(f(y)-x+x) \leq [f(y)-x]f(x)+f(f(x)) = [f(y-x+x)-x]f(x)+f(f(x)) \leq [(y-x)f(x)+f(f(x))-x]f(x)+f(f(x)). $
Ta có $f(f(y))\leq y.f^2(x)+h(x) $ với mọi $y $ (3) ($h(x) $ là hàm xác định từ biểu thức trên).
Chú ý là: từ (1) và (2), ta thấy $0 \leq f(f(x))[f(f(x))-f(x)] $ và $f(x)\leq f(f(x)) $, nên nếu $f(f(x))<0 $ thì phải có $f(f(x))=f(x) $.
Như vậy, từ (3) suy ra: khi $y $ tới âm vô cùng thì $f(y)=f(f(y)) $ cũng tới âm vô cùng.
Thay $y=-x $ vào (*), có $f(0)\leq -xf(x)+f(f(x))=f(x)(-x+1) $ (khi $x $ đủ nhỏ thì $f(x)=f(f(x)) $). Cho $x $ tới âm vô cùng thì vế phải cũng tới âm vô cùng, trong khi vế trái cố định. Mâu thuẫn.
b) Ta chứng minh rằng tồn tại N để với mọi $x<N $ thì $f(x)=0. $
Phản chứng, nếu không tồn tại N thì từ ý a), ta suy ra tồn tại dãy $\{x_n\} $ dần tới âm vô cùng sao cho $f(x_n)<0 $. Như vậy, từ (2) ta suy ra $f(x)-x\leq 0 $.
Lại thay $y=-x $ vào (*), có $f(0) \leq -xf(x)+f(f(x)) \leq -xf(x) $. Chọn dãy $\{x_n\} $ tiến tới âm vô cùng thay vào, ta có vế trái cố định, vế phải ra âm vô cùng. Mâu thuẫn.
c) Xét $x<N $, ta có $0 \geq f(0)=f(f(x)) \geq f(x) =0 $. Vậy $f(0)=0 $.
d) Xét $x<0 $, có $0=f(0)=f(-x+x) \leq -xf(x)+f(f(x))\leq -xf(x) \leq 0 $. Dấu bằng xảy ra khi $f(x)=0 $.
KL: $f(x)=0 $ với mọi $x\leq 0 $.  

 

Bài 4

Giả sử n>0n>0 là một số nguyên. Cho một cái cân hai đĩa và nn quả cân với trọng lượng là 20,21,...,2n−120,21,...,2n−1. Ta muốn đặt lên cái cân mỗi một trong nn quả cân, lần lượt từng quả một, theo cách để bảo đảm đĩa cân bên phải không bao giờ nặng hơn đĩa cân bên trái. Ở mỗi bước ta chọn một trong các quả cân chưa được đặt lên cân, rồi đặt nó hoặc vào đĩa bên trái, hoặc vào đĩa bên phải, cho đến khi tất cả các quả cân đều đã được đặt lên cân. Xác định xem có bao nhiêu cách để thực hiện được mục đích đề ra.

Bài 5

Cho hàm f:Z→Z+f:Z→Z+.Giả sử rằng với hai số nguyên m,nm,n bất kì, hiệu f(m)−f(n)f(m)−f(n) chia hết cho f(m−n).f(m−n).
Chứng minh rằng với mọi số nguyên m,nm,n thỏa mãn f(m)≤f(n)f(m)≤f(n),thì ta có f(n)f(n) chia hết cho f(m)f(m)

Bài 6
Cho tam giác nhọn ABCABC nội tiếp đường tròn ΓΓ.Gọi ll là tiếp tuyến tới TT,và la,lb,lcla,lb,lc là các đường thẳng đối xứng với ll qua BC,CA,ABBC,CA,ABtương ứng.Chứng tỏ rằng đường tròn ngoại tiếp tam giác xác định bởi la,lb,lcla,lb,lc tiếp xúc với đường tròn ΓΓ. 
 

Bài viết đã được chỉnh sửa nội dung bởi Phạm Quang Toàn: 27-07-2011 - 18:07

Bạn copy rồi past lại là có ý gì  ? 




#724117 Giải phương trình: $(x^2+3x+3)\sqrt{2x^2+x+1}-x^3-4x^2-12...

Đã gửi bởi Sin99 on 23-07-2019 - 13:31 trong Phương trình - hệ phương trình - bất phương trình

Bằng Casio và Viet đảo, ta tìm được nhân tử $ x^2  - 3x - 3 $, ta sẽ tách theo nhân tử này. Phương trình tương đương: 

$ (x^2 - 3x -3 )\sqrt{2x^2+x+1} + 6(x+1)\sqrt{2x^2+x+1} - (x+1)(x^2  +3x+9) = 0 $

$ \Leftrightarrow (x^2 - 3x -3 )\sqrt{2x^2+x+1}  + (x+1)( 6\sqrt{2x^2+x+1} - x^2 - 3x - 9 ) = 0 $

$\Leftrightarrow (x^2 - 3x -3 )\sqrt{2x^2+x+1}  + (x+1)[6(\sqrt{2x^2+x+1} - x-2) - x^2 + 3x + 3 ] = 0 $

$ \Leftrightarrow (x^2 - 3x -3 )\sqrt{2x^2+x+1}  + (x+1) [ 6\frac{ x^2 -3x -3}{ \sqrt{2x^2+x+1} + x+2} - (x^2 -3x - 3)] = 0 $

$  \Leftrightarrow  x^2 - 3x -3  = 0 $ hoặc $ \sqrt{2x^2+x+1}  + (x+1)( \frac{6}{ \sqrt{2x^2+x+1} + x+2} - 1 ) = 0 $.

Xử lí vế thứ 2: Đặt $ \sqrt{2x^2 + x+1} = a, x+1 = b $ ta được $ a +b(\frac{6}{a+b+1} - 1) = 0 $ 

Đến đây giải bình thường.




#724115 $\left\{\begin{matrix} y^{2}-2xy=8x^{3}-6x+1 & &...

Đã gửi bởi Sin99 on 23-07-2019 - 12:07 trong Phương trình, hệ phương trình và bất phương trình

Câu c) Hệ tương đương : 

$ \left\{\begin{matrix} \frac{3}{x^2+y^2-1} - 1 = \frac{2y}{x} (1)\\ x^2 + y^2 - 4 = \frac{2x}{y} (2) \end{matrix}\right.$

$ \Leftrightarrow  \left\{\begin{matrix} \frac{4- x^2 - y^2 }{x^2+y^2-1}  = \frac{2y}{x} (1)\\  x^2 + y^2 - 4 = \frac{2x}{y} (2) \end{matrix}\right. $ 

Nhân (1) với (2) theo vế ta được $ \frac{(4-x^2-y^2)^2}{x^2+y^2-1} = -4 $. Đến đây đặt $ x^2 + y^2 = a $ rồi giải bình thường.




#724101 $ \boxed{TOPIC} $ Các bài toán hình học hướng đến Ol...

Đã gửi bởi Sin99 on 22-07-2019 - 23:26 trong Hình học phẳng

Mình thêm giả thiết tam giác ABC nhọn rồi đó.

Nhọn hay tù mình vẽ hình vẫn không ra, trừ phi bạn nói $ d $ đi qua  điểm  $ O $ 




#724098 $ \boxed{TOPIC} $ Các bài toán hình học hướng đến Ol...

Đã gửi bởi Sin99 on 22-07-2019 - 22:22 trong Hình học phẳng

AP cắt $(I)$ tại K. Q đối xứng với K qua AM. FP cắt $(I)$ tại $R$

Do $AM.AI=AK.AP=AD^2$ nên KMIP nội tiếp. Do đó $\widehat{KMP}=\widehat{KIP}$ suy ra $\widehat{KMP}+\widehat{KMA}+\widehat{AMQ}=\widehat{KIP}+2\widehat{KPI}=180^0$ nên P, M, Q thẳng hàng.

Do vậy nên ta có: $\frac{DM}{MP}=\frac{ME}{MP}=\frac{QE}{DP}=\frac{DK}{DP}=\frac{DA}{PA}=\frac{DA}{FP}$ (1)

Mặt khác,do F đối xứng với A qua DP nên DP là phân giác $\widehat{RPK}$ suy ra $DR=DK=QE$ và $DE=RQ$

Do đó: $\widehat{RPM}=\widehat{ADM}$ (2)

Từ (1) và (2) suy ra tam giác FPM đồng dạng với tam giác ADM nên $\widehat{FMP}=\widehat{AMD}=90^0$.

Bài 2: Cho tam giác $ABC$ và $(O)$ là đường tròn ngoại tiếp. Một đường thẳng d bất kỳ đi qua $(O)$ cắt $AB,AC$ lần lượt tại $M$ và $N$. Gọi $I,P,Q$ lần lượt là trung điểm của $MN,BN,CM$. Chứng minh 4 điểm $O,I,P,Q$ cùng nằm trên một đường tròn.

P/s: Ai biết xuất xứ bài toán này không cho mình biết với....

 

Bạn xem lại đề bài 2, mình thấy có vấn đề vì khi vẽ có thể rơi vào trường hợp O,I,P,Q là tứ giác lõm ... 




#724051 $ \boxed{TOPIC} $ Các bài toán hình học hướng đến Ol...

Đã gửi bởi Sin99 on 22-07-2019 - 02:08 trong Hình học phẳng

$ \boxed{ \text{Giới thiệu}} $ Chào các bạn, khát khao của mình sau khi đậu chuyên là sẽ tích cực hoạt động ở VMF, và cho đến bây giờ, mục tiêu thứ nhất xem như đã hoàn thành, đến với mục tiêu thứ 2  :icon6: Để nhằm hiện thực khát khao ấy, cũng như nâng cao kiến thức, kĩ năng giải hình học phẳng, mình quyết định mở 1 Topic nhỏ với đối tượng hướng đến các bạn đam mê hình học, có nguyên vọng chinh phục các kì thi HSG ở bậc THPT từ lớp 10 đến lớp 12.

 

$ \boxed{ \text{Nội dung}} $ Như đã nói như trên, topic sẽ xoay quanh các bài toán hình học phẳng qua các mức độ dễ đến khó. Mỗi các nhân sẽ post 1 bài toán đề nghị do sưu tầm, sáng tác,... với điều kiện đưa ra lời giải của bài toán trước đó. Topic sẽ không giới hạn số lời giải của 1 bài toán. Các bạn sau khi post lời giải cần post bài toán đề nghị và cứ như thế... đến khi đơm hoa kết trái, mình sẽ tổng hợp thành file và lưu lại sau này. Do đây mới là topic nhỏ nên độ khó của các bài toán cũng ở mức giới hạn, dành cho cả HS vừa vào lớp 10.  :D Topic này được mình tham khảo từ Topic Marathon số học Olympic  :wub:

 

$ \boxed{\text{Qui định}} $

1) Các bài toán được ghi nguồn rõ ràng

2) Nội dung xoay quanh hình học phẳng, nhằm luyện tập hướng đến các kì thi Olympic.

3) Lời giải rõ ràng, cụ thể, không lan man,...

4) Không được đăng các bài toán vẫn đang trong thời gian diễn ra của những cuộc thi khác ( VD: Toán học và tuổi trẻ, toán tuổi thơ,...)

5) Bạn giải bài thứ $ n $ thì đề xuất luôn bài toán thứ  $ n +1 $

6) Tuyệt đối không spam, quảng cáo, mình sẽ báo cáo các anh quản trị viên có hình phạt thích hợp.

7) Sau 3 ngày bài toán không có lời giải, người đề cử sẽ đăng bài toán đó và tiếp tục đề cử bài mới. 

 

(P/s) Do đây là lần đầu, cộng thêm kiến thức còn rất hạn hẹp, nên có sai sót gì mong các anh chị, các bạn bỏ qua, cùng nhau đóng góp để topic và cá nhân mình ngày càng hoàn thiện.   :icon6:

 

Mình xin đề xuất bài toán đầu tiên: 

$ \boxed{\text{Bài toán 1}} $ (Gặp gỡ Toán học 2010)

Đường tròn $ (I) $ nội tiếp tam giác $ ABC $ tiếp xúc các cạnh $ AB, AC $ tương ứng tại  $ D , E $. $ P $ là 1 điểm bất kì trên cung lớn $ \mathop DE $ của $ (I) $. Lấy $ F $ là điểm đối xứng với $ A $ qua $ PD $ và $ M $ là trung điểm đoạn $ DE $. Chứng minh rằng $ \angle FMP  = 90^{\circ} $. 

Hình gửi kèm

  • Untitled.png



#723991 CMR \sum ab \leq 27

Đã gửi bởi Sin99 on 20-07-2019 - 14:58 trong Bất đẳng thức và cực trị

Cho các số thực $ a,b,c  \geq 2 $ thỏa mãn điều kiện $ a+b+c = \frac{1}{a} + \frac{1}{b} + \frac{1}{c} + 8 $.

CMR  $ \sum ab \leq 27 $ 




#723990 Phương pháp giải phương trình nghiệm nguyên bằng phương pháp sử dụng công thứ...

Đã gửi bởi Sin99 on 20-07-2019 - 14:41 trong Số học

Đặt $ x^3 = a $, phương trình tương đương $ a^2 + (a- y)^2 = 320 $ . 




#723963 Chứng minh

Đã gửi bởi Sin99 on 19-07-2019 - 22:33 trong Hình học

Áp dụng định lí Cosin , ta có : 

$ \frac{AC^{2}+CB^{2}-BA^{2}}{CB^{2}+BA^{2}-AC^{2}} = \frac{ 2AC.BC.Cos C }{ 2AB.BC.Cos B } = \frac{ AC. Cos C}{ AB. Cos B } = \frac{ AC. \frac{KC}{AC} }{ AB. \frac{ KB}{AB} } = \frac{ KC}{KB} $ (ĐPCM)

( Định lí Cosin thì bạn có thể google, chứng minh chỉ bằng lượng giác và biến đổi đại số )  :ukliam2:




#723962 Trại hè Phương Nam

Đã gửi bởi Sin99 on 19-07-2019 - 22:19 trong Thi HSG cấp Tỉnh, Thành phố. Olympic 30-4. Đề thi và kiểm tra đội tuyển các cấp.

Ban co the giai thich cho minh sao gia thiet lai co b-c dong du 15(mod 31).Minh cam on

Từ giả thiết suy ra $ a +b - c - ( a - b + c - 1) $ chia hết cho 31 hay $ 2(b-c) + 1 $ chia hết cho 31, suy ra $ 2(b-c) \equiv  30 $ (mod 31 ) do $ ( 2,31)  = 1 $




#723960 Trại hè Phương Nam

Đã gửi bởi Sin99 on 19-07-2019 - 22:15 trong Thi HSG cấp Tỉnh, Thành phố. Olympic 30-4. Đề thi và kiểm tra đội tuyển các cấp.

Câu 4 a)

Theo định lí Viet cho phương trình bậc 3 ta có : 

  $\left\{\begin{matrix} b+ x_{1}+ x_{2}= -2a(1)\\ bx_{1} + bx_{2} + x_{1}x_{2} = 2a^2 +b (2)\\ bx_{1}x_{2} = -c (3) \end{matrix}\right.$

Từ (1) $ \Rightarrow b^2+ x_{1}^2+ x_{2}^2 + 2(bx_{1} + bx_{2} + x_{1}x_{2}) = 4a^2$ 

$ \Leftrightarrow  b^2+ x_{1}^2+ x_{2}^2 + 4a^2 + 2b = 4a^2 $ 

$ \Leftrightarrow  b^2+ x_{1}^2+ x_{2}^2 = 0 $. Suy ra $ b^2 + 2b \leq 0 $ hay $ -2 \leq b \leq 0 $ Vậy $ |b| \leq 2 $.




#723958 Trại hè Phương Nam

Đã gửi bởi Sin99 on 19-07-2019 - 22:00 trong Thi HSG cấp Tỉnh, Thành phố. Olympic 30-4. Đề thi và kiểm tra đội tuyển các cấp.

Câu 3:

Từ giả thiết suy ra  $b -c \equiv 15 $ (mod 31) $ \Rightarrow a \equiv 16 $ (mod 31).

Ta có $0 \equiv ab -c -1 \equiv 16(c+15) -c -1 \equiv 15c + 15.16- 1 = 15c + 22$ (mod 31)

 $\Rightarrow 15c \equiv 9$ (mod 31) $ \Rightarrow 5c \equiv 3$ (mod 31) (do $ (3,31) =1$). Tồn tại 1 số nguyên dương k để $  31.k  + 3 $ chia hết cho 5 và $ 1 \leq k \leq 4 $. Khi đó $ 5c \equiv 3$ (mod 31) $ \Leftrightarrow $ $ 5c  \equiv  31k  + 3 $ (mod 31). Chọn $ k = 2 $ ta được $ c  \equiv 13 $ (mod 31) $ \Rightarrow  b  \equiv  28 $ (mod 31).

Vậy $ a + bc  \equiv  16 + 13.28  \equiv 8 $ (mod 31).




#723941 số học

Đã gửi bởi Sin99 on 19-07-2019 - 17:25 trong Số học

Bạn ghi lại rõ đề, $ 2^{2^{n-1}} $ hay $ 2^{2^n -1 } $ 




#723938 Phương pháp giải phương trình nghiệm nguyên bằng phương pháp sử dụng công thứ...

Đã gửi bởi Sin99 on 19-07-2019 - 17:16 trong Số học

Bạn xem lại đoạn <=>  3^y = (x+1)^2 - 9 + 1 = (x+1-3)(x+1+3) + 1  = (x-2)(x+4) + 1  suy ra x đồng dư 1 mod 3 ? 





#723937 $ | \overrightarrow{OP_{1}}+ \overrightarrow{OP_{2}} +...+...

Đã gửi bởi Sin99 on 19-07-2019 - 17:09 trong Bất đẳng thức và cực trị

Cho $ 2n +1 $ điểm, kí hiệu là $ P_{i} $ trên nửa đường tròn tâm $ O $ bán kính bằng 1 cm. CMR: $ | \overrightarrow{OP_{1}}+ \overrightarrow{OP_{2}} +...+\overrightarrow{OP_{2n+1}} | \geq 1$. 




#723936 Own inequality ?

Đã gửi bởi Sin99 on 19-07-2019 - 17:06 trong Bất đẳng thức và cực trị

$\sum \frac{a^2}{b^2+2ab} = \sum \frac{\frac{a^2}{b^2}}{1+2\frac{a}{b}} \geq \frac{(\sum \frac{a}{b})^2}{3+2\sum \frac{a}{b} }$.

Đặt $ \sum \frac{a}{b} = x $ , ta chứng minh $ \frac{x^2}{3+2x} \geq \frac{x}{3} $ hay BĐT $ \Leftrightarrow x(x-3) \geq 0 $ (Đúng).

Ta chỉ cần chứng minh $ \sum \frac{a}{b} \geq \sqrt{3(a^2+b^2+c^2)} $ hay $ \sum \frac{a^2}{b^2} + 2 \sum \frac{b}{a} \geq 3(a^2+b^2+c^2) $. 

Ta có : $ \frac{a^2}{b^2} + \frac{a}{c} + \frac{a}{c} \geq 3 \sqrt[3]{ \frac{a^4}{b^2c^2} } = 3 \frac{a^2}{ \sqrt[3]{a^2b^2c^2} } $ Tương tự cộng theo vế, ta được $ (\sum \frac{a}{b})^2 \geq \frac{3( a^2+b^2+c^2)}{ \sqrt[3]{a^2b^2c^2} } $ hay $ \sum \frac{a}{b} \geq \sqrt{3(a^2+b^2+c^2)} $ do $ abc =1$.

Chứng minh hoàn tất!  :D  :lol:




#723845 $\left\{\begin{matrix} y^{2}-2xy=8x^{3}-6x+1 & &...

Đã gửi bởi Sin99 on 17-07-2019 - 09:55 trong Phương trình, hệ phương trình và bất phương trình

$ \left\{\begin{matrix} x^3+y^3 +xy =2x + 4y -1 (1)\\ xy+ x+ 2y =1 (2) \end{matrix}\right. $

(1) + (2)x2 : $ x^3 + y^3 + 3xy = 1 \Leftrightarrow x^3 + y^3 - 1^3 = + 3.(-1).xy \Leftrightarrow (x+ y - 1 )(x^2+y^2+1 -xy + x+y ) = 0 $




#723829 Phương pháp giải phương trình nghiệm nguyên bằng phương pháp sử dụng công thứ...

Đã gửi bởi Sin99 on 16-07-2019 - 22:17 trong Số học

Câu c) mình giải thế này. Ta có đánh giá $ x^2  + xy + y^2 = (x+y)^2 - xy \geq (x+y)^2 - \frac{(x+y)^2}{4} = \frac{3(x+y)^2}{4} $. 

$ Pt \Leftrightarrow \frac{39}{7} = \frac{x^2+xy+y^2}{x+y} \geq \frac{3(x+y)^2}{4(x+y)} = \frac{3}{4}(x+y) \Rightarrow \frac{52}{7} \geq x+ y $. Do $ x,y  \in Z , x  +y > 0 $ nên $ 7 \geq x+y \geq 1 $. Từ đó thế x theo y để giải phương trình bậc 2.




#723828 Phương pháp giải phương trình nghiệm nguyên bằng phương pháp sử dụng công thứ...

Đã gửi bởi Sin99 on 16-07-2019 - 22:10 trong Số học

Câu b) thì đánh giá $ x,y $ nhờ xét $ \Delta $ và giải bất phương trình, nhưng có cách nào thu hẹp $ y $ lại hơn không nhỉ  :(




#723827 Phương pháp giải phương trình nghiệm nguyên bằng phương pháp sử dụng công thứ...

Đã gửi bởi Sin99 on 16-07-2019 - 22:03 trong Số học

Mình đóng góp lg theo cách của bạn 

B1 a)

$ x^2 + 2y^2 + 3xy +3x+5y  = 15 $ 

$ \Leftrightarrow x^2 + 3(y+1)x + 2y^2  + 5y - 15 = 0 $

$ \Rightarrow \Delta =  y^2 - 2y + 69 $. Để phương trình có nghiệm nguyên thì $ \Delta $ phải là số chính phương. Đặt $ y^2 - 2y + 69 = k^2  \Rightarrow  (k + y - 1)(k -y  +1) = 68 $.  Từ đó dễ tìm được  $x,y \in Z $.




#723826 Cho a, b, c: độ dài cạnh tam giác, $0\leq m\leq 1, CM:\su...

Đã gửi bởi Sin99 on 16-07-2019 - 21:55 trong Bất đẳng thức và cực trị

$ \sum \sqrt{ \frac{a}{b+c - ma } } = \sum \frac{a}{ \sqrt{a(b+c-ma)} } \geq \sum 2\frac{a}{a+b+c-ma}   = \sum \frac{a^2}{a^2+ab+ac-ma^2} \geq \frac{(a+b+c)^2}{(a+b+c)^2 - m(a^2+b^2+c^2)} \geq \frac{3(a+b+c)^2}{(3-m)(a+b+c)^2} = \frac{3}{3-m} $ Ta cần cm $ \frac{3}{3-m} \geq \sqrt{m+1}$. Thật vậy, 2 vế dương nên BĐT tương đương 

$ \frac{9}{(m-3)^2} \geq m+1 $. Ta có $ VT \geq \frac{9}{4} , VP \leq 2 $ Vậy có dpcm. Dấu "=" không xảy ra. 




#723824 Chứng Minh Biểu Thức

Đã gửi bởi Sin99 on 16-07-2019 - 21:37 trong Phương trình, hệ phương trình và bất phương trình

Ta có $ 1 + 2 + ...+x = \frac{(x+1)x}{2} , 1^2+2^2+...+x^2 = \frac{x(x+1)(2x+1)}{6} \Leftrightarrow (1 + 2 + ...+x )(1^2+2^2+...+x^2)  = \frac{x^2(x+1)^2(2x+1)}{12} $

Lần lượt xét số dư của $x$ cho 3 ta cm được $ \frac{x^2(x+1)^2(2x+1)}{12} \in  Z $ và có vô số $ x $ nguyên để $ (2x+1) $ là số chính phương, từ đó có dpcm




#723823 Chứng Minh Biểu Thức

Đã gửi bởi Sin99 on 16-07-2019 - 21:26 trong Phương trình, hệ phương trình và bất phương trình

a) Không mất tính tổng quát giả sử $ x \leq y \leq z$. Khi đó ta có $ \frac{1}{1991}= \frac{1}{x} + \frac{1}{y} + \frac{1}{z} \leq \frac{3}{x} \Leftrightarrow x \leq 3.1991 $ Như vậy nghiệm x thuộc đoạn $ [1,3.1991]$ là hữu hạn nên có dpcm.




#723800 Bài toán thú vị cho học sinh giỏi

Đã gửi bởi Sin99 on 15-07-2019 - 23:39 trong Đại số

Bài này đề chuyên sư phạm mà :v

$ Pt \Leftrightarrow \frac{x^2- 4x\sqrt{x-1} + 4(x-1)}{x} + \frac{y^2- 4y\sqrt{y-1} + 4(y-1)}{y} = 0 $

$ \Leftrightarrow \frac{(x-2\sqrt{x-1})^2}{x} + \frac{(y-2\sqrt{y-1})^2}{y} = 0 $




#723757 Bất đẳng thức

Đã gửi bởi Sin99 on 14-07-2019 - 10:20 trong Bất đẳng thức và cực trị

BĐT $ \Leftrightarrow \frac{b-c}{3a-b+c} \geq 0 \Leftrightarrow \frac{(b-c)^2}{(b-c)(3a-b+c)} \geq 0 $ ( Đúng theo Cauchy Schwars)